Find Triplets of Positive Integers with Sum of Cubes

In summary, the conversation was about the importance of communication in relationships. The speakers discussed how open and honest communication can lead to a stronger and healthier relationship, while lack of communication can lead to misunderstandings and conflict. They also emphasized the importance of actively listening and validating each other's feelings in order to foster a deeper understanding. Ultimately, the conversation highlighted the crucial role that communication plays in maintaining successful relationships.
  • #1
anemone
Gold Member
MHB
POTW Director
3,883
115
Find all triples (a, b, c) of positive integers such that ##a^3+b^3+c^3=(abc)^2##.
 
  • Like
Likes topsquark
Physics news on Phys.org
  • #2
Its as easy as one, two, three
if you write a program to see,
but could be harder to find,
if not so inclined.

Python:
for a in range(100):
    for b in range(a,100):
        for c in range(b,100):
            
            abc = a * b * c
            abc2 = abc * abc
            a3 = a * a * a
            b3 = b * b * b
            c3 = c * c * c
            sum = a3 + b3 + c3

            if sum == abc2:
                print("a=%d / b=%d / c=%d\n"%(a,b,c))
 
Last edited:
  • Haha
Likes bob012345
  • #3
Where's your proof that there are no solutions outside of your search range?
 
  • Like
Likes bob012345 and malawi_glenn
  • #4
anemone said:
Find all triples (a, b, c) of positive integers such that ##a^3+b^3+c^3=(abc)^2##.
ambiguity alert: Are a,b, and c required to be different positive integers? I assume not.
 
Last edited:
  • #5
Pr
PAllen said:
Where's your proof that there are no solutions outside of your search range?

No proof here. @anemone asked for three numbers which I took to be three distinct numbers.

However, by inspection one can see that if the numbers were 0, 1, n where n can vary from 1 to infinity the left side value will increase much faster than the right side value implying that the answer must be in small numbers.

Also to be solvable it must be limited to what we can do manually on paper or mentally in our heads.

Being a robot, I initially searched on a much larger range but then realized that 1-100 would be sufficient and not give away the solution. Being a mentor, I tried to disguise my solution like Penn and Teller on Fool Us in clever banter or maybe not so clever banter.
 
  • Like
Likes anemone
  • #6
The trivial solution is 0, 0, 0. This is the only solution where one of the integers are 0.
 
  • Sad
Likes PeroK
  • #7
Positive excludes 0. Allowing that would be nonnegative.
 
  • Like
Likes anemone and PeroK
  • #8
jedishrfu said:
@anemone asked for three numbers
It reads "find ALL"
 
  • #9
The solution set is not obviously bounded. If ##a=b## and ##c=1##, then the right hand side grows faster.
 
  • Like
Likes PeroK
  • #10
PAllen said:
Where's your proof that there are no solutions outside of your search range?
Let c be variable x
[tex]f(x):=x^3-a^2b^2x^2+a^3+b^3[/tex]
The extremes are
[tex]f(0)=a^3+b^3>0[/tex]
and
[tex]f(\frac{2a^2b^2}{3})=a^3+b^3-\frac{2^2}{3^3}a^6b^6=-\frac{2^2}{3^3}(a^3-\frac{3^3}{2^2} )(b^3-\frac{3^3}{2^2})+ \frac{3^3}{2^2}<0[/tex]
so that f(x)=0 has positive solutions in the range of
[tex]0<c_1<\frac{2a^2b^2}{3}<c_2[/tex]
Say a<b<c with obvious 1<a
[tex]a<c<\frac{2b^2c^2}{3}[/tex]
[tex]b<c<\frac{2c^2a^2}{3}[/tex]
But this inequality fails to give upper limit of a, b, c.

[EDIT] For the case of a=1,b=2
[tex]f(x)=(x-3)(x^2-x-3)[/tex]
So c=3.
I am not sure whether there are positive integer solutions other than this {1,2,3}.
If we allow them to be zero or negative, {0,0,0},{-a,0,a} and {-1,1,1} are the obvious solutions.
 
Last edited:
  • Like
Likes anemone
  • #11
PAllen said:
ambiguity alert: Are a,b, and c required to be different positive integers? I assume not.
That wasn't stated but it is easy to show whether that must be the case or not. Assume they are the same and see if the real root is integer or not.
 
Last edited:
  • #12
jedishrfu said:
Pr

No proof here. @anemone asked for three numbers which I took to be three distinct numbers.

However, by inspection one can see that if the numbers were 0, 1, n where n can vary from 1 to infinity the left side value will increase much faster than the right side value implying that the answer must be in small numbers.

Also to be solvable it must be limited to what we can do manually on paper or mentally in our heads.

Being a robot, I initially searched on a much larger range but then realized that 1-100 would be sufficient and not give away the solution. Being a mentor, I tried to disguise my solution like Penn and Teller on Fool Us in clever banter or maybe not so clever banter.
Your partial solution and poem are definitely amusing but the above argument is wholly inadequate. Even if you ban duplicates, the following is true: ##for~every~c \gt 3, there~exist~triples~a\lt b\lt c## such that sum of cubes is greater than the product of squares, and other triples where it is less. Thus no matter how large c is, there is crossover region. Once the problem is properly solved, I can give a simple formula for generating near equality triples for arbitrary c. In problems like this, where all solutions are requested, the proper demonstration of that is typically where 'all the real math' is.
 
  • Like
Likes jedishrfu, PeroK and bob012345
  • #13
I took it that ##a,b,c≥1##. They can't be zero. It is also easy to show if they could be negative there would be an infinite number of triples.
 
  • #14
Office_Shredder said:
The solution set is not obviously bounded. If ##a=b## and ##c=1##, then the right hand side grows faster.
But there is no (integer) real root thus no solution in terms of integer triples in that case for any ##a##.
 
Last edited:
  • #15
Just checking policy here - my understanding was that no science advisors or homework helpers should provide a solution until a valid solution was posted by someone else (I see no issue with hints and commentary). Is this still the policy expected here? I would think it should be, but just asking.
 
Last edited:
  • #17
Given no new activity, I'll post some more possibly useful hints. Note, I have not succeeded in solving this - twice I thought I had argument for why no more solutions, and twice I exploded the argument.

The way to find 'near solutions' that I referred to is to note, assuming ##a\le b \le c##, that ##a^2b^2=c## implies that the sum of cubes is greater than the product of squares. Generally, for any a,b as close as possible (from below) this condition (or matching it) increasing either a or b by 1 means that the product of squares is greater than or equal to the sum of cubes. More rigorously, for c above some small minimum, it is easy to show that ##a^2b^2=2c## forces the product of squares to be larger. Note, all of this is just algebra.

From here, I also looked at what can be concluded by treating this as an polynomial in c. Then, since the cubic coefficient is 1, the rational roots theorem directly gives criteria for integer roots. Playing around with this I could conclude that a positive integer root required that factors of an expression of cubes of a and b must make another expression involving squares of a and b and the candidate root a perfect square. However I could not see any way to argue from this how there is a finite number of solutions. Perhaps some other approach is needed.

Empirically, after not having programmed in 6 years, and never having seen python before, I downloaded an installation of it and developed the following program from results I derived. It searches by increasing c, and is ##O(c^{1.25})## instead of brute force ##O(c^{3})##. Forgive any neophyte features, as this is literally the first day I touched python.

Code:
d=0
rat=10
for c in range(4,10000001):
    for a in range(1,int(c**.25)+1):
        b=int(c**.5/a)+1
        while 1:
            L=a**3 + b**3 + c**3
            R=(a*b*c)**2
            if R==L: d=1
            nrat=R/L    
            if nrat<rat:
                rat=nrat
                sa=a
                sb=b
                sc=c
            if R >= L : break
            b=b+1
        if d==1: break
    if d==1: break
print(sa,sb,sc)
print(rat)

Using this, varying the search limits (the lower limit must be greater than 2 for proper behavior), I find no solutions beyond (1,2,3) up to c=10 million. However, what I do find is that the larger limit of c you search, the closer you get to a solution. With lower search bound of 4, the following show best case up to indicated upper bound:

Up to 100:
1,10,99 ratio of R/L = 1.01 (rounded)
Up to 1000
1,31,960 ratio of R/L = 1.001 (rounded)
Up to 10000
1,100,9999 ratio R/L = 1.0001 (rounded)
Up to 100,000
1,316,99855 ratio R/L = 1.00001 (rounded)
Up to 1,000,000
1,1000,999999 ratio R/L = 1.000001 (rounded)
Up to 10,000,000
1,3162,9998243 ratio R/L = 1.0000001 (rounded)

(At this point, it seemed the next large order magnitude search would take over an hour, and I don't have that patience. I'm sure my code is not the fastest way to do this in Python.)

I hope this helps someone else solve the problem, but I admit I am stuck at this point.
 
Last edited:
  • Like
Likes anemone, PeroK and bob012345
  • #18
[tex]a^3+b^3+c^3=(abc)^2...(1)[/tex]
integers
[tex]0<a \leq b \leq c[/tex]
[tex]a^3+b^3=(a+b)(a^2-ab+b^2) \equiv 0 (mod\ c^2)...(2)[/tex]

a+b < 2c because a=b=c does not satisfy formula (1)

If a+b##\neq##c, [tex]a^2-ab+b^2=a(a-b)+b^2 < c^2 [/tex] (2) is not satisfied.

So a+b=c, [tex]a^2-ab+b^2=c^2-3ab\equiv 3ab \equiv 0(mod\ c)[/tex]
Let 3ab=kc with a positive integer k, (1) gives
[tex]k^2c^2-18c+9k=0[/tex]
as a quadratic equation of c
[tex]D=18^2-36k^2 \ge 0[/tex]
[tex]9\ge k^2[/tex]
k=1,2,3 are candidates.
k=1: [tex]c^2-18c+9=0[/tex]
[tex]c=9\pm 6\sqrt{2}[/tex] not an integer
k=2:
[tex]2c^2-9c+9=0[/tex]
[tex](2c-3)(c-3)=0[/tex]
Thus a=1,b=2,c=3 is a set of integer solution
k=3:
c=1 but a=b=c=1 does not satisfy (1).

So {1,2,3} is a unique set of solution.

[EDIT] Take a look at my post #21 for the incompleteness of the above.
 
Last edited:
  • Like
Likes dextercioby and anemone
  • #19
@anuttarasammyak , there is an early point on your argument I don't follow. You have a+b<2c, and another factor ##<c^2##. However, suppose the first is 1.5c, and the other is ##2/3 c^2##. Then your equation 2 would be satisfied. I don't see how you exclude cases like this.
 
Last edited:
  • Like
Likes PeroK and anuttarasammyak
  • #20
@anuttarasammyak , I wonder if you could use my result with your argument, that a solution must have ##c\lt a^2b^2\lt 2c##, except for very low c, which could be handled explicitly.
 
  • #21
The formula (2) in post #18 gives the cases of
Case 1
[tex]a+b\equiv 0(mod\ c^2)[/tex]
Case 2
[tex]a^2-ab+b^2 \equiv 0(mod\ c^2)[/tex]
Case 3
[tex]a+b\equiv 0(mod\ p)[/tex],
[tex]a^2-ab+b^2 \equiv 0(mod\ q)[/tex]
where ##pq=c^2## where p is a devisor of c.

Only case 3 is possible because
Case 1
[tex]a+b<2c \leq c^2[/tex]
Case 2
[tex]a^2-ab+b^2 \leq b^2 \leq c^2[/tex] where two equals, i.e. a=b=c which does not satisfy (1) is excluded. so ##a^2-ab+b^2 < c^2##

As for case 3, in my post #8 I made
[tex]p=q=c[/tex]
Now I find that other cases should be investigated for possible c of non prime number.
 
Last edited:
  • #22
My attempt is here.

Given ##a<b<c##, let ##a=n+1## where ##n≥0##

$$(n+1)^3 + b^3 +c^3 = (n+1)^2b^2c^2$$ expanding

$$1 + 3n^2 +3n +n^3 + b^3 + c^3 = b^2c^2(n^2 + 2n + 1)$$ expanding
$$1 + 3n^2 +3n +n^3 + b^3 + c^3 = n^2b^2c^2 + 2nb^2c^2 + b^2c^2$$ regrouping

$$n^3 + b^3 + c^3 -(nbc)^2 = b^2c^2(2n + 1) -3n^2 -3n -1$$ again

$$n^3 + b^3 + c^3 -(nbc)^2 = nb^2c^2 +b^2c^2(n + 1) -3n(n+1) -1$$ finally

$$n^3 + b^3 + c^3 -(nbc)^2 = nb^2c^2 + (n+1)(b^2c^2-3n) -1$$

Notice that the LHS is just our original equation with ##n## instead of ##a##. Also, notice that the RHS is always positive since ##b^2c^2>3n## yet the LHS can be negative or positive. Demanding ##n=0## removes the inconsistency thus the smallest integer is ##a=1##.

Now we have

$$1 + b^3 + c^3 =b^2c^2$$ apply the same logic to ##b## letting ##b=n+2## ##n≥0##.

$$1 + (n+2)^3 + c^3 = (n+2)^2c^2$$ expanding

$$1 + n^3 +3(2)n^2 + 3(2^2)n + 2^3 + c^3 = (n^2 + 4n +4)c^2$$ regrouping

$$1 + n^3 + c^3 - n^2c^2= 4(n +1)c^2 -6n^2 -12n -8$$ again

$$1 + n^3 + c^3 - (nc)^2= 4c^2 + 4nc^2 -6n^2 -12n -8$$ since ##c>n+2##

$$1 + n^3 + c^3 - (nc)^2= 4c^2 + 4n(n+2)^2 -6n^2 -12n-8$$
$$1 + n^3 + c^3 - (nc)^2= 4c^2 + 4n^3 + 10n^2 + 4n - 8$$

The same arguments can be made as before so we must demand ##n=0## and ##b=2## which forces ##c=3##.

[\SPOILER]
 
Last edited:
  • Like
Likes dextercioby
  • #23
anuttarasammyak said:
The formula (2) in post #18 gives the cases of
Case 1
[tex]a+b\equiv 0(mod\ c^2)[/tex]
Case 2
[tex]a^2-ab+b^2 \equiv 0(mod\ c^2)[/tex]
Case 3
[tex]a+b\equiv 0(mod\ p)[/tex],
[tex]a^2-ab+b^2 \equiv 0(mod\ q)[/tex]
where ##pq=c^2## where p is a devisor of c.
I’m not sure even this can be required without further justification. You could have c=42, p=63, q=28. p is < 2c, q < ##c^2##, both are factors of ##c^2##, but neither are factors of c. And, of course, the product is ##c^2##. There may well be an argument to exclude this possibility, but it has to be given
 
  • Like
Likes PeroK and anuttarasammyak
  • #24
bob012345 said:
My attempt is here.

Given ##a<b<c## let ##a=n+1## where ##n≥0##

$$(n+1)^3 + b^3 +c^3 = (n+1)^2b^2c^2$$ expanding

$$1 + 3n^2 +3n +n^3 + b^3 + c^3 -n^2b^2c^2 = b^2c^2(n^2 + 2n + 1)$$
You've lost me here. Where does ##-n^2b^2c^2## come from?
 
  • #25
PAllen said:
I’m not sure even this can be required without further justification. You could have c=42, p=63, q=28. p is < 2c, q < c2, both are factors of c2, but neither are factors of c. And, of course, the product is c2. There may well be an argument to exclude this possibility, but it has to be given
Thanks. p is a divisor of c^2, not of c, my bad.
For the case of non-prime c, more work should be done.
 
Last edited:
  • #26
PAllen said:
You've lost me here. Where does ##-n^2b^2c^2## come from?
Sorry, I corrected a double counting of that term but it comes from the first term of $$b^2c^2(n^2 + 2n + 1)$$ from the RHS just moved to the LHS. It should not have been on the LHS at that point and has been corrected. Thanks.
 
  • #27
bob012345 said:
$$n^3 + b^3 + c^3 -(nbc)^2 = nb^2c^2 + (n+1)(b^2c^2-3n) -1$$

Notice that the LHS is just our original equation with ##n## instead of ##a##. Also, notice that the RHS is always positive since ##b^2c^2>3n## yet the LHS can be negative or positive. Demanding ##n=0## removes the inconsistency thus the smallest integer is ##a=1##.
This argument is not valid as stated. All you have shown is that n=0 is part of a possible solution. The fact that many choices of n,b,c don't work, and some make the LHS negative, proves nothing. You need an argument that no such choices work, if n>0.
 
Last edited:
  • Like
Likes PeroK
  • #28
bob012345 said:
Notice that the LHS is just our original equation with ##n## instead of ##a##. Also, notice that the RHS is always positive since ##b^2c^2>3n## yet the LHS can be negative or positive. Demanding ##n=0## removes the inconsistency thus the smallest integer is ##a=1##.
I don't see the inconsistency here. ##n = a-1## is fixed. It's not a free variable.
 
  • #29
Thanks everyone for the overwhelming participation. I want to encourage for those who have "partially" solved the question right but cannot prove those are the unique answers to have further attempt and I wish you success in finding the proof.

However, here is one brilliant solution of someone that I want to share:
WLOG, let ##a\ge b \ge c##. Note that

##(abc)^2 \le 3a^3 \implies a\ge \dfrac{(bc)^2}{3}##

On the other hand, we have

##a^2 \mid b^3+c^3 \implies a^2\le2b^3##

Combining both bounds gives ##\dfrac{(bc)^4}{9}\le 2b^3 \implies bc^4\le 18##. Thus, ##c=1## and ##b\le 18##.

Check the remaining cases manually, the answers are the six permutations of ##(3, \, 2,\,1)##.
 
  • Like
  • Love
Likes Opalg, dextercioby, bob012345 and 5 others
  • #30
anemone said:
However, here is one brilliant solution of someone that I want to share:
WLOG, let ##a\ge b \ge c##. Note that

##(abc)^2 \le 3a^3 \implies a\ge \dfrac{(bc)^2}{3}##

On the other hand, we have

##a^2 \mid b^3+c^3 \implies a^2\le2b^3##

Combining both bounds gives ##\dfrac{(bc)^4}{9}\le 2b^3 \implies bc^4\le 18##. Thus, ##c=1## and ##b\le 18##.

Check the remaining cases manually, the answers are the six permutations of ##(3, \, 2,\,1)##.
At the risk of making myself look stupid (not hard), could someone kindly explain something from the proof in the Post #29 Spoiler.

It seems that we jump from ##a\ge b \ge c## to ##(abc)^2 \le 3a^3##.

But this inequality is not always true. For example:
##a=4, b=3, c=2##
##(abc)^2 = (4 \times 3 \times 2)^2= 576##
##3a^3 = 3 \times 4^3 = 192##
 
  • #31
Steve4Physics said:
At the risk of making myself look stupid (not hard), could someone kindly explain something from the proof in the Post #29 Spoiler.

It seems that we jump from ##a\ge b \ge c## to ##(abc)^2 \le 3a^3##.
You combine that inequality with ##(abc)^2 = a^3 + b^3 + c^3 \le 3a^3##.
 
  • Like
  • Informative
Likes anemone, malawi_glenn, topsquark and 1 other person
  • #32
anemone said:
However, here is one brilliant solution of someone that I want to share:
Smart! Thanks for the sharing.

I would like to add some observation. We can introduce
[tex]a=Ag,b=Bg,c=Cg[/tex]
where g is common divisor among the three and A,B,C are coprime to others. The original problem is expressed as
[tex]A^3+B^3+C^3=(ABC)^2g^3[/tex]
By this answer g=1 which brings the original formula and now we know a, b and c are coprime. And
[tex]b<a<b^2c^2[/tex]
Those may reduce candidates of a and b to be tested to some extent.
 
Last edited:
  • Skeptical
Likes PeroK
  • #33
anuttarasammyak said:
I would like to add some observation. We can introduce
[tex]a=Ag,b=Bg,c=Cg[/tex]
where g is common devisors among the three and A,B,C are coprime to others.
Not necessarily: ##a = 2, b = 6, c = 15## have no common factor, but they are not pairwise coprime.
anuttarasammyak said:
The original problem is expressed as
[tex]A^3+B^3+C^3=(ABC)^2g^3[/tex]
By this answer g=1
Why?
 
  • #34
PS it's true that if ##a, b, c## is a solution, then ##ka, kb, kc## is not a solution for ##k > 1##.
 
  • #35
PeroK said:
I don't see the inconsistency here. ##n = a-1## is fixed. It's not a free variable.
The only condition I assumed is that ##a<b<c## so it is not fixed.
 
Last edited:

Similar threads

  • Math POTW for Secondary and High School Students
Replies
14
Views
1K
  • Math POTW for Secondary and High School Students
Replies
1
Views
786
  • Math POTW for Secondary and High School Students
Replies
1
Views
766
  • Math POTW for Secondary and High School Students
Replies
1
Views
1K
  • Math POTW for Secondary and High School Students
Replies
1
Views
830
  • Math POTW for Secondary and High School Students
Replies
1
Views
796
  • Math POTW for Secondary and High School Students
Replies
3
Views
900
  • Math POTW for Secondary and High School Students
Replies
4
Views
1K
  • Math POTW for Secondary and High School Students
Replies
1
Views
892
  • Math POTW for Secondary and High School Students
Replies
2
Views
766
Back
Top